Bài tập đề nghị & gợi ý– đáp số

Một phần của tài liệu Sách Chuyên đề số học (Trang 133 - 149)

Hệ thặng dư và định lý Thặng dư Trung Hoa

6.4 Bài tập đề nghị & gợi ý– đáp số

Do đó từ (6.16) và (6.17), với lưu ý rằng x≡0 (mod 10), suy ran là số tự nhiên thỏa mãn các điều kiện đề bài khi và chỉ khin= x

10, vớix

là số nguyên thỏa mãn hệ điều kiện sau:

           x≡0 (mod 10) x≡1 (modq1) x≡r2 (modq2) 0≤x <10q1q2 r1, r2 ∈ {0;−4;−5} (6.18)

Vì10; q1; q2 đôi một nguyên tố cùng nhau nên theo định lí Thặng dư Trung Hoa, hệ (6.18) có nghiệm duy nhất.

Dễ thấy sẽ có 9sốxlà nghiệm của9 hệ (6.18) tương ứng. Vì mỗi sốx

cho ta một sốnvà hai sốx cho hai sốnkhác nhau nên có9 sốnthỏa

mãn các điều kiện đề bài.

Nhận xét. Ví dụ 6.19 chính là trường hợp đặc biệt của bài toán tổng quát sau:

Ví dụ 6.20. Cho số nguyên dương n có phân tích tiêu chuẩn n =

pα1

1 pα2

2 ...pαk

k . Xét đa thứcP(x)có hệ số nguyên. Nghiệmx0 của phương trình đồng dưP(x)≡0 (mod n) là lớp đồng dưx0 ∈

0,1,2, ..., n−1

thỏa mãnP(x0)≡0 (mod n). Khi đó, điều kiện cần và đủ để phương trình P(x) ≡ 0 (mod n) có nghiệm là với mỗi i = 1,2, ..., s, phương trình P(x) ≡ 0 (modpαi

i ) có nghiệm. Hơn nữa, nếu với mỗi i = 1,2, ..., s, phương trình P(x) ≡ 0 (mod pαi

i ) có ri nghiệm module pαi

i

thì phương trình có r=r1r2...rs nghiệm modulen. △

6.4 Bài tập đề nghị & gợi ý – đáp số

Bài tập đề nghị

Bài1. a. Chứng minh rằng: Nếu (a, m) = 1 và x chạy qua một hệ thặng dư đầy đủ modulo m thì ax+b, với b là một số nguyên tùy ý, cũng chạy qua một hệ thặng dư đầy đủ modulem.

126 6.4. Bài tập đề nghị & gợi ý – đáp số

b. Chứng minh rằng: Nếu(a, m) = 1 và x chạy qua một hệ thặng dư thu gọn modulom thìaxcũng chạy qua một hệ thặng dư thu gọn modulem.

Bài2. Mỗi số nguyên dươngT được gọi là số tam giác nếu nó có dạng

T = k(k+ 1)

2 , trong đóklà một số nguyên dương. Chứng minh rằng tồn tại một HĐĐ modulengồmn số tam giác.

Bài3. a. Chom1, m2 là hai số nguyên dương nguyên tố cùng nhau. Chứng minh rằng:

Φ(m1m2) = Φ(m1).Φ(m2)

b. Giả sử số nguyên dương m có phân tích chính tắc thành tích các thừa số nguyên tốm=pα1 1 pα2 2 ...pαk k . Chứng minh rằng: Φ(m) =pα1−1 1 pα2−1 2 ...pαkk−1(p1−1)(p2−2)...(pk−1) Bài4. Tính tổng sau: S= 2012 X k=6 17k 11

Bài5. Cho số nguyên dươngnvà số nguyên tốplớn hơnn+ 1. Chứng minh rằng đa thức P(x) = 1 + x n+ 1 + x2 2n+ 1+...+ xp pn+ 1

không có nghiệm nguyên.

Bài6. Cho p là số nguyên tố có dạng 3k+ 2 (k nguyên dương). Tìm số dư khi chiaS =

p

P

k=1

(k2+k+ 1)cho p.

Bài7. Cho các số nguyên dươnga, bthỏa mãn(a, b) = 1. Chứng minh rằng phương trình ax+by = 1 có vô số nghiệm nguyên (x, y)

6.4. Bài tập đề nghị & gợi ý – đáp số 127 Bài8. Tìm số nguyên dương nhỏ nhất có tính chất: chia 7 dư5, chia

11 dư7, chia13 dư 3.

Bài9. Chứng minh rằng tồn tại một dãy tăng{an}∞n=1các số tự nhiên sao cho với mọi số tự nhiên k, dãy{k+an} chỉ chứa hữu hạn các số nguyên tố.

Bài10. Số nguyên dươngn được gọi là có tính chấtP nếu như với các số nguyên dương a, b mà a3b+ 1...n thì a3+b...n. Chứng minh rằng số các số nguyên dương có tính chấtP không vượt quá24.

Bài11. Tìm tất cả các số tự nhiênnthỏa mãn2n−1chia hết cho3 và có một số nguyênm mà 2n−1

3 |4m2+ 1.

Bài12. Chứng minh rằng tồn tại số tự nhiên k sao cho tất cả các số

k.2n+ 1 (n= 1,2, ...) đều là hợp số.

Gợi ý – đáp số

Bài1. Chứng minh trực tiếp dựa vào định nghĩa.

Bài2. Ta chứng minh n phải có dạng n = 2k. Phản chứng, giả sử

n= 2k.m với m lẻ và m > 1. Sử dụng tính chất hệ thặng dư đầy đủ.

Bài3. Ta có thể chứng minh dựa vào kiến thức về hệ thặng dư đầy đủ, cũng có thể chứng minh dựa vào định lí Thặng dư trung Hoa.

Bài4. Sử dụng HTG.

Bài5. Biểu diễnP(x)dưới dạngP(x) =apxp+ap−1xp−1+...+a2x2+

a1x+a0. Phản chứng, giả sử P(x) có nghiệm nguyên x = u. Suy ra mâu thuẫn.

Bài6. Tiến hành tương tự Ví dụ6.7.

Bài7. Sử dụng kiến thức HĐĐ.

128 6.4. Bài tập đề nghị & gợi ý – đáp số Bài8. Đáp số: 887.

Bài9. Gọi pk là số nguyên tố thứ k, k > 0. Theo định lí Thặng dư Trung Hoa, tồn tại dãy số {an}∞n=1 thỏa mãn a1 = 2;an =

−k(modpk+1),∀k≤n.

Bài10. Định lý Thặng dư Trung Hoa.

Bài11. Chứng minh n có dạng 2k. Sử dụng tính chất của số Fecma (xem lại Ví dụ6.15).

Chương 7 Một số bài toán số học hay trên VMF 7.1 m3+ 17...3n 129 7.2 c(ac+ 1)2= (5c+ 2)(2c+b) 136

Phần này gồm một số bài toán hay được thảo luận nhiều trên Diễn đàn Toán học. Bạn đọc có thể vào trực tiếp topic của bài toán đó trên Diễn đàn Toán học, bằng cách click vào tiêu đề của bài toán đó.

7.1 m3 + 17...3n

Bài toán 7.1. Chứng minh rằng với mọi số nguyên dương n, tồn tại một số tự nhiên m sao cho

m3+ 17...3n

. △

Đầu tiên, chúng ta đến với chứng minh đề xuất cho bài toán đầu bài.

Chứng minh. Ta sẽ chứng minh bài toán bằng quy nạp. Vớin= 1, ta chọnm= 4.

Vớin= 2, ta chọnm= 1.

Giả sử bài toán đúng đếnn=k, hay ∃m∈N:m3+ 17...3k

Ta chứng minh rằng đối với trường hợpn=k+ 1cũng đúng tức là tồn tại một sốm′ sao cho m′3+ 17...3k+1.

Đặtm3+ 17 = 3k.n⇒n6...3.

130 7.1. m3+ 17...3n ⇒ n≡2 n≡1 (mod3)⇒ m3+ 17≡2.3k m3+ 17≡3k mod3k+1 • Trường hợp 1: m3+ 17≡2.3k (mod 3k+1) Xét: (m+ 3k−1)3 =m3+m23k+m32k−1+ 33k−3≡m3+m23k (mod 3k+1) (Dok≥2⇒32k−1...3k+1 và 33k−3...3k+1). Suy ra: m+ 3k−13+ 17≡m3+m2.3k+ 17≡2.3k+m2.3k≡0 (mod 3k+1) (vìm6...3⇒m2≡1 (mod 3)⇒2 +m2...3⇒(2 +m2).3k...3k+1). Như vậy, ở trường hợp 1, ta có: m+ 3k−13

+ 17...3k+1. • Trường hợp 2: m3+ 17≡3k (mod 3k+1). Xét: m−3k−13=m3−m23k+m32k−1−33k−3 ≡m3−m23k (mod 3k+1) (Dok≥2⇒32k−1...3k+1 và 33k−3...3k+1). Suy ra: m−3k−13+ 17≡m3−m23k+ 17≡3k−m23k≡0 (mod 3k+1) (vìm6...3⇒m2≡1 (mod 3)⇒1−m2...3⇒ 1−m2 .3k...3k+1). Như vậy, ở trường hợp 2 ta có: m−3k−13

+ 17...3k+1. Tóm lại, ta đều tìm được số nguyên t6...3 màt3+ 17...3k+1.

Ta đã chứng minh được vấn đề đúng trong trường hợpn=k+ 1. Theo nguyên lý quy nạp, ta có đpcm.

7.1. m3+ 17...3n 131 Bổ đề 7.1– Cho a, b, qlà các số nguyên thỏa(a;q) = 1 và q >0. Khi ấy, luôn tồn tạik∈Z sao choak±b...q.

Chứng minh. Ta chứng minh đại diện cho trường hợpak+b...q. Trường hợp còn lại tương tự.

XétA={1; 2; 3;...;q}là 1 hệ đầy đủ HĐĐ mod q.

Theo tính chất của Hệ thặng dư, ta có tậpB={a; 2a; 3a;...;qa} cũng là HĐĐ mod q.

⇒C={a+b; 2a+b; 3a+b;...;qa+b} cũng là HĐĐ mod q.

Do đó, tồn tạik∈[1;q]sao cho ak+b...q.

Nhận xét. Bài toán đã cho thực chất là yêu cầu tìm 1 sốx nguyên sao chox+ 17...3n vàx là lập phương 1 số nguyên. Bổ đề trên đã cho thấy sự tồn tại của x nguyên để x+ 17...3n. Còn việc tìm x để là x là lập phương 1 số nguyên thì ta sẽ dùng phương pháp quy nạp như trên. Đối với 1 người yêu toán, ta phải không ngừng sáng tạo. Ta hãy thử tổng quát bài toán đã cho:

• thay vì m3, ta thử thaymk vớik là số nguyên dương cố định.

• thay vì 3n, ta thử thaypn vớip là 1 số nguyên tố.

• thay số17 bởi y∈Nvới y cố định.

Kết hợp các thay đổi trên, ta có 1 bài toán "tổng quát" hơn

Dự đoán 7.1– Cho p là số nguyên tố. y, k∈Nvà y, k cố định. Khẳng định hoặc phủ định mệnh đề sau

∀n∈N,∃x∈N:xk+y...pn (7.1) Ta thử thay một vài giá trịp, k, yvào để thử xem (7.1) có đúng không. Khi thayk= 2, y= 1, p= 3 thì mệnh đề (7.1) trở thành

∀n∈N,∃x∈N:x2+ 1...3n (7.2)

132 7.1. m3+ 17...3n

Rất tiếc, khi này, (7.2) lại sai!!!. Ta sẽ chứng minh (7.2) sai khin≥1. Thật vậy, để chứng minh dự đoán7.1 sai, ta cần có bổ đề sau

Bổ đề 7.2– Cho plà số nguyên tố dạng 4k+ 3vàa, b∈Z. Khi đó a2+b2...p⇔(a...p)∧(b...p)

Từ (7.2), suy ra x2+ 1...3. Áp dụng bổ đề 7.2với p= 3, ta suy ra 1...3: vô lý.

Vậy khin≥1thì 6 ∃x∈Z:x2+ 1...3n.

Không nản lòng, ta thử thêm một vài điều kiện để (7.1) trở nên chặt hơn và đúng. Nếu bạn đọc có ý kiến nào hay, xin hãy gửi vào topicnày

để thảo luận. Sau khi thêm một số điều kiện, ta có 1 bài toán hẹp hơn nhưng luôn đúng.

Định lý 7.1– Cho p nguyên tố lẻ. y, k∈Nvà y, k cố định. Biết rằng gcd(k, p) =gcd(k, p−1) =gcd(y, p) = 1.

Chứng minh rằng:

∀n∈N,∃x∈N:xk+y...pn (7.3)

Chứng minh. Trước hết, để chứng minh (7.3), ta cần có bổ đề sau

Bổ đề 7.3– Cho plà số nguyên tố lẻ.k nguyên dương thỏa

(k;p) = (k−1;p) = 1

Khi đó, {1k; 2k;...; (p−1)k}là HTG modp.

Chứng minh. Gọig là căn nguyên thủy củap tức làordp(g) =p−1. Khi đấy thìg1, g2, ..., gp−1 lập thành 1 HTG modp và rõ ràng

ga1, ga2, ..., gap−1 là HTG modp⇔a1, a2, .., ap−1 là HĐĐ của p−1. Với 1 ≤ i ≤ p−1 thì tồn tại ai để mà i ≡ gai (mod p) và rõ ràng

ai lập thành 1 HTG modp nên hệ 1k,2k, ..,(p−1)k có thể viết lại là

gk, g2k, ..., g(p−1)k, nó là HTG modpkhi và chỉ khik,2k, ...,(p−1)k là hệ thặng dư đầy đủ củap−1, tức là knguyên tố cùng nhau với p−1.

7.1. m3+ 17...3n 133

Quay lại bài toán. Ta chứng minh (7.3) bằng phương pháp quy nạp. Vớin= 1, theo bổ đề7.3thì

∃x0 ∈ {1; 2;...;p−1}:x0k≡ −y (modp)⇒xk0+y...p

Giả sử bài toán đúng đếnnhay tồn tạixk+y...pn

Ta sẽ chứng minh n+ 1cũng đúng hay tồn tại xk

0+y...pn+1

Thật vậy, từ giả thiết quy nạp suy raxk+y=pn.q • Trường hợp 1: q...p⇒ đpcm • Trường hợp 2: gcd(q, p) = 1 (7.4) Khi đó ta chọnx0 =v.pn+x Do đó xk0+y= (v.pn+x)k+y =vk.pnk+ 1 k .vk−1.pn(k−1).x+...+ k−1 k .v.pn.xk−1+ (xk+y) (7.5) Dễ dàng chứng minh pn+1|vk.pnk+ 1 k .vk−1.pn(k−1).x+... k−2 k .v2.p2n.xk−2 Do vậy ta xét k−1 k .v.pn.xk−1+ (xk+y) =k.v.pn.xk−1+pn.q=pn(k.v.xk−1+q)

Nhận thấy giả sửk.xk−1 ≡t (modp) mà gcd(k, p) = 1 và xk+y...p⇒ gcd(x, p) = 1 (dogcd(y, p) = 1) suy ragcd(t, p) = 1

Do đó(k.v.xk−1+q)≡tv+q (modp)mà từ (7.4) ta đã cógcd(q, p) = 1

Cho nên luôn tồn tại vthỏa mãn tv+q...p. Do đó bài toán được khẳng đinh vớin+ 1.

Theo nguyên lý quy nạp, bài toán đã được chứng minh.

134 7.1. m3+ 17...3n

Chưa dừng lại ở đây, nếu trong (7.3), ta thaykbởix, ta sẽ được 1 bài toán khác:

Định lý 7.2– Cho p nguyên tố lẻ. y ∈ N và y cố định. Biết rằng gcd(y, p) = 1. Khi đó:

∀n∈N,∃x∈N:xx+y...pn (7.6)

Chứng minh. Ta chứng minh bài toán này bằng phương pháp quy

nạp. Ta coi định lý 7.1 như 1 bổ đề. Dễ thấy nếu x thỏa (7.6) thì

gcd(x;p) = 1.

Khi đó, vớin= 1, ta xét hệ đồng dư (I)

x≡k (mod (p−1))

x≡x0 (modp)

trong đó, x0;k∈N thỏaxk

0 +y...p.

Dogcd(p−1;p) = 1 nên theo định lý Thặng dư Trung Hoa thì hệ (I) luôn có nghiệmx′.

Chọnx=x′, ta chứng minh xthỏa (7.6) khi n= 1. Thật vậy

gcd(x;p) = 1⇒xp−1≡1 (modp)⇒xk≡xx (modp)

⇒xx+y≡xk+y ≡xk

0+y≡0 (modp)

Vậy∃x∈N:xx+y...p.

Giả sử (7.6) đúng đến n−1, tức là tồn tạix0 đểx0x0 +y...pn−1. Theo cách chứng minh quy nạp ở (7.6), ta chọn được xn = apn+x0

thỏaxx0

n +y...pn.

Khi đó, dễ nhận thấyxn≡x0 (modpn−1). Ta xét hệ đồng dư (II)

X≡x0 (mod (pn−1(p−1)))

X≡xn (modpn)

Dogcd(pn−1(p−1);pn) = 1nên theo định lý Thặ ng dư Trung hoa, hệ (II) có nghiệmX. Ta chứng minh x=X thỏa (7.6). Thật vậy

7.1. m3+ 17...3n 135

Mặt khácXx0

≡xnx0 (mod pn) (do cách chọn trong hệ (II)).

⇒XX+y≡xnx0 +y≡0 (modpn)

Theo nguyên lý quy nạp, bài toán đã được chứng minh.

Mở rộng của bài toán đầu đề vẫn còn nhiều, như tăng thêm điều kiện để chặn như(m3+ 17...3n)∧(m3+ 176...3n+1), v.v. Rất mong nhận được ý kiến đóng góp cho việc mở rộng.

Lời cảm ơn

Rất cảm ơnNguyen Lam Thinh,Karl Heinrich Marx,nguyenta98,The Gunnerđã đóng góp ý kiến và mở rộng cho bài viết này.

136 7.2. c(ac+ 1)2 = (5c+ 2)(2c+b)

7.2 c(ac+ 1)2 = (5c+ 2)(2c+ b)

Bài toán 7.2. Cho 3 số nguyên dương a;b;c thoả mãn đẳng thức: c(ac+ 1)2 = (5c+ 2b)(2c+b) (7.7)

Chứng minh rằng : c là số chính phương lẻ. △

Nhận xét. Thoạt nhìn vào bài toán, thật khó để tìm 1 phương pháp cho loại này. Nhận xét trong giả thiết ở VP (7.7), thì b xuất hiện với bậc là2. Thế là ta có 1 hướng nghĩ là dùng tam thức bậc 2 cho bài toán này. Ta không nên chọn c vì bậc của c là 3, không chọna vì phương trình mới theoahiển nhiên trở lại (7.7)

Chứng minh (Chứng minh 1). c(ac+ 1)2 = (5c+ 2b) (2c+b) ⇔2b2+ 9bc+ 10c2−c(ac+ 1)2 = 0 ∆b = 81c2−4.2.10c2−c(ac+ 1)2=c2+ 8c(ac+ 1)2 ⇒∆b =chc+ 8 (ac+ 1)2i=x2,(x∈N∗) Đặt d=GCD(c;c+ 8(ac+ 1) 2)⇒d|8 (ac+ 1)2 d|c⇒(ac+ 1)2;d= 1 ) ⇒d|8 • Trường hợp 1: d=8 ⇒c 8; c 8+ (ac+ 1) 2 = 1 chc+ 8 (ac+ 1)2i=x2(x∈N)⇔ 8c.c 8+ (ac+ 1) 2 =x 8 2 ⇒8|x⇒x= 8x2(x2 ∈N∗)⇒ c8.c 8 + (ac+ 1) 2=x22 ⇒    c 8 =t 2 c 8 + (ac+ 1) 2 =p2 t;p∈N∗ (t;p) = 1 ⇒ c= 8t2 t2+ 8t2a+ 12 =p2 Mà dễ chứng minh 8t2a+ 12 < t2+ 8t2a+ 12 < 8t2a+ 22 ⇒ 8t2a+ 12 < p2 < 8t2a+ 22 : mâu thuẫn

7.2. c(ac+ 1)2= (5c+ 2)(2c+b) 137 Do đó,d= 8 bị loại. • Trường hợp 2: d=4 ⇒c 4; c 4+ 2 (ac+ 1) 2 = 1 ⇒ 4c;c 4+ 2 (ac+ 1) 2 là những số chính phương (*) Nếu c 4 là số chẵn ⇒ c 4+ 2 (ac+ 1) 2...2 ⇒4c;c 4 + 2 (ac+ 1) 2= 2: mâu thuẫn. Do đó, c 4 là số lẻ. Mà c 4 là số chính phương ⇒ c 4 ≡1 (mod 4) Mặt khác, do cchẵn nên ac+ 1là số lẻ ⇒(ac+ 1)2 ≡1 (mod 4) ⇒ 4c + 2 (ac+ 1)2 ≡1 + 2.1≡3 (mod 4): vô lý do (*). Do đó,d= 4 bị loại. • Trường hợp 3: d=2 . Tương tự tự Trường hợp 2, ta có c 2 lẻ ⇒ c 2 ≡1 (mod 8) cchẵn nên ac+ 1lẻ⇒(ac+ 1)2 ≡1 (mod 8) ⇒ c2+ 4(ac+ 1)2 ≡1 + 4.1≡5 (mod 8) : vô lý Do đó,d= 2 bị loại. • Trường hợp 4: d=1

Tương tự trường hơp 2, ta có ngay c lẻ và do (c;c+ 8(ac+ 1)2) = 1

nên clà số chính phương.

Vậy ta có đpcm.

Nhận xét. Ta thấy trong bài này, b và c có 1 mối liên quan khá chặt chẽ với nhau nên ta thử giải theob, c sử dụng kĩ thuậtGCD tức là đặt

Một phần của tài liệu Sách Chuyên đề số học (Trang 133 - 149)